User avatar
 
noah
Thanks Received: 1192
Atticus Finch
Atticus Finch
 
Posts: 1541
Joined: February 11th, 2009
 
This post thanked 2 times.
 
 

Re: Q17 - A body of circumstantial

by noah Fri Dec 31, 1999 8:00 pm

This is a tough problem. The conclusion of the argument is that a body of evidence is still strong even if a few parts of it are discredited? Why, because it's like a rope in the fact that a few broken strands of a rope do not render a rope broken.

When an LSAT argument relies on an analogy, you want to check out the applicability of the analogy. In this case, perhaps an argument is not like a rope -- are all the parts of an argument equally important? Perhaps it's more like a rope with a steel cable weaved in -- the cable is more important to the rope's strength than the rope fibers. Let's say that one piece of evidence -- that the suspect was not in the country -- is the keystone of a defense. Discrediting that may completely destroy a defense, even if there are some other pieces of evidence. (A) describes this potential problem.

Many of the wrong answers are wrong because the lawyer simply does not do what the answer choice suggests:
(B) is incorrect because the argument does not assume that a body of evidence is less than (or more than) the sum of its parts.
(C) is incorrect because the argument does in fact consider this -- and says that it would not matter!
(D) is wrong because the lawyer does suggest that an argument and a rope have a similarity -- that's the point!
(E) is incorrect because the lawyer does not restate a claim as a conclusion -- he or she uses an analogy to make a point.

Does that clear it up?


#officialexplanation
 
nzheng
Thanks Received: 0
Forum Guests
 
Posts: 10
Joined: May 18th, 2010
 
 
 

Q17 - A body of circumstantial

by nzheng Tue May 25, 2010 12:05 pm

I was down to three choices- A, C and D, and ended up selecting C. Really would like some help on understanding why it's A,thanks.
 
goriano
Thanks Received: 12
Atticus Finch
Atticus Finch
 
Posts: 113
Joined: December 03rd, 2011
 
 
 

Re: Q17 - A body of circumstantial...

by goriano Sat Dec 03, 2011 3:19 pm

I'm still very confused by your explanation in eliminating (C).

(C) is incorrect because the argument does in fact consider this -- and says that it would not matter!


While the conclusion does state that it wouldn't matter (even if a few items of a body of circumstantial evidence are discredited, the overall body retains it's basic strength), I thought the purpose of a "fails to consider" answer choice is to bring up an alternative explanation to the conclusion. That is, that discrediting items WOULD discredit the overall body of evidence.

As an analogy, say, if the author's conclusion was "X causes Y," and an answer choice stated "fails to consider that Y causes X," that would presumably be the correct answer even though applying your explanation for (C) you would argue "well, the conclusion DOES consider this and says that it would not matter."

Am I missing something here?
 
jpchris3
Thanks Received: 1
Jackie Chiles
Jackie Chiles
 
Posts: 31
Joined: September 15th, 2010
 
 
 

Re: Q17 - A body of circumstantial

by jpchris3 Fri May 25, 2012 6:29 pm

I think (C) is incorrect because it states "if many iterms in a body of circumstantial evidence ..., while the lawyer is only making an argument concerning at most "a few: items of a body of evidence
 
nflamel69
Thanks Received: 16
Atticus Finch
Atticus Finch
 
Posts: 162
Joined: February 07th, 2011
 
 
 

Re: Q17 - A body of circumstantial

by nflamel69 Sat Sep 01, 2012 4:22 pm

This question is similar to the question with the tv production and the absence rate of its workers (can't remember the question). both are essentially original act wouldn't be affected if something is being taken out of it. But for that question, similar explanation to answer A didn't work, instead we had to address the absence rate. So how come on this one this explanation worked? Could one of the geeks clarify this?
 
patrice.antoine
Thanks Received: 35
Atticus Finch
Atticus Finch
 
Posts: 111
Joined: November 02nd, 2010
 
 
 

Re: Q17 - A body of circumstantial

by patrice.antoine Wed Sep 12, 2012 11:37 am

BUMP!

Confused with this one. I see where (A) works but I'm not quite sure where (C) is restated in the argument. The argument's analogy equivocates "one strand" with "a few items"...

...unless "a few" in LSAT world means at least one?

UGH. I think I see my error! :o :oops:
 
hippo3717
Thanks Received: 1
Forum Guests
 
Posts: 25
Joined: October 12th, 2012
 
 
 

Re: Q17 - A body of circumstantial

by hippo3717 Sat Oct 20, 2012 3:55 pm

Can someone help me deciphering answer choice A?

I think it means that author makes an unwarranted assumption that circumstantial evidence isn't the "most" strong evidence present among other ones: after all, it if were to be the most important evidence and get discredited, then whole entire argument may fall apart.
User avatar
 
ohthatpatrick
Thanks Received: 3808
Atticus Finch
Atticus Finch
 
Posts: 4661
Joined: April 01st, 2011
 
This post thanked 6 times.
 
 

Re: Q17 - A body of circumstantial

by ohthatpatrick Mon Oct 22, 2012 9:25 pm

Hey, there.

Let me try to answer some of the lingering questions/qualms.

When flaw answer choices begin with "takes for granted", "presumes w/o justification", or "assumes without warrant", you can evaluate whether the rest of the answer is correct or not by asking yourself, "Did the author NEED to assume this?"

Basically, those three phrases turn the idea that follows into the answer to a Necessary Assumption question. So when I see "takes for granted", "presumes", etc., I evaluate the phrase that follows just like I would a Nec. Assump question, including a deep suspicion of extreme language and a willingness to apply the Negation Test.

(A) is definitely something the author needs to assume to keep his analogy intact.

The author was assuming that each strand of a rope and each piece of circumstantial evidence is roughly equally load-bearing. If we negate (A), it says "some items in a body of circumstantial evidence ARE significantly more critical to the strength of the evidence than other items in that body".

That would weaken the lawyer's argument. As Noah was saying before, it's possible that opposing counsel could discredit one (or more) of the most critical pieces of circumstantial evidence, thereby greatly weakening the overall body of evidence.

(A) is NOT comparing circumstantial evidence to other types of evidence.

(A) is dealing with the issue of this question: does every piece of circumstantial evidence contribute roughly equally to the overall body of evidence?

====

When a Flaw answer choice begins with "fails to consider", "neglects the possibility", "ignores the possibility", you can evaluate whether the rest of the answer is good by asking yourself, "if true, would this idea weaken the argument?"

So let's do that with (C).

Pretend you're answering a Weaken question for this argument and (C) says:

(C) If many items in a body of circumstantial evidence were discredited, the overall body of evidence would be discredited.

Does that weaken the argument?

No. The author's conclusion is that a few items being discredited wouldn't significantly affect the strength of the overall body.

The author would probably agree with (C) ... the author thinks the more pieces that are discredited, the more the body of evidence is weakened.

When we're trying to argue with a conclusion that is essentially a conditional statement, we need to find an objection that accepts the first half but denies the second half.

So we want an objection that would say "it's possible that only a few items of a body would be discredited, and the overall body of evidence would NOT retain its basic strength."

(A) gives us this objection: if some pieces of evidence are significantly more important than others, then it's possible that discrediting a few pieces COULD do major damage to the overall body of evidence.

In terms of what these quantity words specifically mean, "a few" generally means 3 or 4.

"Many" is not a specific quantity. In most LSAT questions, we are wise to treat "many" like "some" (because we do NOT want to equate "many" with "most").

But clearly, in normal usage, "many" and "a few" are not equivalent terms, so (C) is outside the scope of the author's conclusion.

(Total tangent: "few" is not the same as "a few". If I say "a few of my friends like Green Day", then there are a handful of friends I have who like Green Day. If I say "few of my friends are zombies", it just means less than half of my friends are zombies ... it doesn't necessarily mean that I really DO have any zombie friends. "Few" = a minority, just as "most" = a majority. "Most" is compatible with "all". "Few" is compatible with "none". "A few" is NOT compatible with "none".)

Hope this helps.
 
nflamel69
Thanks Received: 16
Atticus Finch
Atticus Finch
 
Posts: 162
Joined: February 07th, 2011
 
 
 

Re: Q17 - A body of circumstantial

by nflamel69 Tue Mar 19, 2013 11:07 pm

so why can't we equate a few with many? and what kind of quantifier would we equate a few to?
 
timsportschuetz
Thanks Received: 46
Elle Woods
Elle Woods
 
Posts: 95
Joined: June 30th, 2013
 
 
trophy
First Responder
 

Re: Q17 - A body of circumstantial

by timsportschuetz Wed Nov 13, 2013 12:13 am

Answer choice (C) is wrong for the following reason: the argument concludes that "Even if....blalblalba". You have to notice that EVEN IF is NOT a conditional indicator and thus we cannot infer any conditional logic from this statement! This is a common LSAT trap. Since the conclusion has no conditional logic and therefore we cannot make any valid logical inferences, any answer choice that restates the terms from the original argument's conclusion in a valid conditional logic form CANNOT impact our argument - therefore, these types of answer choices are wrong!

Again, "EVEN IF" does not provide us with any conditional logic!
 
vivianabro
Thanks Received: 0
Vinny Gambini
Vinny Gambini
 
Posts: 4
Joined: October 03rd, 2010
 
 
 

Re: Q17 - A body of circumstantial

by vivianabro Tue Dec 03, 2013 6:44 pm

Tim,
I got this question right but I am curious about your post because it may help with future questions. You stated:
"Since the conclusion has no conditional logic and therefore we cannot make any valid logical inferences, any answer choice that restates the terms from the original argument's conclusion in a valid conditional logic form CANNOT impact our argument - therefore, these types of answer choices are wrong!"
I have never heard this before. Why are you saying that an answer stated in logical terms/with logical language cannot impact a conclusion when that conclusion is not stated in logical terms? Would you mind elaborating or providing an example. I am at the moment looking at other questions to try to understand/validate what you said but would love to get more details.
Thank you
 
judaydaday
Thanks Received: 6
Jackie Chiles
Jackie Chiles
 
Posts: 40
Joined: January 14th, 2015
 
 
 

Re: Q17 - A body of circumstantial

by judaydaday Tue May 19, 2015 11:31 pm

I just want to clarify the distinctions you made between a few and few.

I'm not sure what you mean by: "A few" is NOT compatible with "none". Does that mean "a few" means some? But that doesn't seem to make with your explanation. Or is it a matter of degree?

few < a few < many
 
jasonleb1
Thanks Received: 1
Jackie Chiles
Jackie Chiles
 
Posts: 32
Joined: April 09th, 2015
 
 
 

Re: Q17 - A body of circumstantial

by jasonleb1 Sun Nov 08, 2015 1:29 am

I'm still confused by C. It seems to me that Noah is saying " a few" and "many" DO mean the same thing, while Patrick is saying they do not. In either case, I'm not seeing how him saying that a few of the pieces of evidence being discredited still leaves the overall body of evidence intact addresses the possibility that his overall body will fall apart if many are discredited.
User avatar
 
ohthatpatrick
Thanks Received: 3808
Atticus Finch
Atticus Finch
 
Posts: 4661
Joined: April 01st, 2011
 
This post thanked 1 time.
 
 

Re: Q17 - A body of circumstantial

by ohthatpatrick Mon Nov 09, 2015 10:21 pm

A couple things:
1. "few" vs. "a few" is not in any way important to LSAT

2. The whole point of distinguishing "a few" from "many" is just to make the point that (C) could not possibility weaken the conclusion, since the conclusion makes a claim about scenarios in which only a few items are discredited. (C) makes a claim about scenarios in which many items are discredited.

You said "I'm not seeing how 'a few' addresses the claim about 'many'." Exactly. :)

They are two different claims. That's (one reason) why (C) is wrong.

Suppose an author said,
"I've been studying LSAT for months and my top practice test score has been a 145. My target school is Harvard, which has an average score of 172. Thus, even if I get a 150, I still won't get into Harvard."

Can we fault that author for
"failing to consider the possibility that if he were to get a 180, he WOULD get into Harvard?"

No, not at all. He's claiming that with a 150 he wouldn't get into Harvard. If we're going to disagree with his conclusion, we have to show him a way in which people with 150's DO get into Harvard.

Similarly, to disagree with THIS author's conclusion, we have to show him a way in which ONLY A FEW pieces of circumstantial evidence are discredited, but the evidence HAS lost its basic strength.

(A), the correct answer, is in Necessary Assumption form. If we negated it, we'd be saying "SOME items in a body of circumstantial evidence ARE significantly more critical to the strength of the evidence." This is an objection, because it shows a way where ONLY A FEW discredited pieces of evidence WOULD compromise the basic strength of the evidence.


=== to clarify the completely unimportant quantity word discussion from before ===

Correct, "a few" is not compatible with "none".

If it's true that "A few of my friends have met the President", then it is false that "none of my friends have met the President".

The visual of
few < a few < many
is correct

but, again, COMPLETELY unimportant.

"Few" is an adjective. It means 'less than half', but no one in the real world uses it when the word "none" would otherwise be applicable.

Let's say that only one of my friends has ever met the President. Which of these are true?

Some of my friends have met the President.
Few of my friends have met the President.
A few of my friends have met the President.
Many of my friends have met the President.

Only the 1st and 2nd are true, because the 3rd and 4th require "more than one" in order to be true.

Hope this helps.
User avatar
 
LolaC289
Thanks Received: 21
Elle Woods
Elle Woods
 
Posts: 92
Joined: January 03rd, 2018
 
 
 

Re: Q17 - A body of circumstantial

by LolaC289 Sun Aug 19, 2018 4:34 am

I think it's a very interesting question. I was left with (A) & (C) also, found (C) super strange because it looks like a total repeat of the Lawyer's own conclusion, but I thought (A) was too strong and need not to be assumed. Now I see why (A) is correct although the Lawyer doesn't seem to be assuming this. "A few items" from the whole are totally randomly extracted and can apply to each and every item in the whole set. In order to make this claim universally correct, we have to make sure there aren't some "special items" that carries extraordinary weight and destroys the whole set which destroys the claim.

For me, Noah's explanation for (C) made sense to me. I think if it changed from "FAILS TO CONSIDER THAT (taking some items out)...(the whole) would BE discredited" to "TAKES FOR GRANTED THAT (taking some items out)...(the whole) would NOT be discredited", it would be correct. Flaws are basically in two forms: illegally assuming things & forget to consider some other things, here the author DID consider if many/a few were discredited, it's just he thought it would be fine. So he didn't forget to consider the things that (C) mentions.